Cardinality of an interval as a limit

In summary: When $Ie [a, b]$, there are no rationals in the interval $[a, b]$. So $|I \cap A_n|$ is either $\infty$ or the number of rationals in the interval $[a, b]$. Since $\infty$ is not a member of $A_n$, $|I \cap A_n|$ is $\infty$.
  • #1
Fermat1
187
0
Let $I$ be an interval and $A_{n}$ be the set of $k/n$ where $k$ is an integer.

Prove that $|I|$ is the limit as $n$ tends to infinity of $\frac{1}{n}|(IA_{n})|$ where $IA_{n}$ denotes intersection.

My plan was to split it up into cases for the different type of intervals and come up with formulas for $|(IA_{n})|$, but I'm finding that very tricky.

Thanks
 
Physics news on Phys.org
  • #2
Fermat said:
Let $I$ be an interval and $A_{n}$ be the set of $k/n$ where $k$ is an integer.

Prove that $|I|$ is the limit as $n$ tends to infinity of $\frac{1}{n}|(IA_{n})|$ where $IA_{n}$ denotes intersection.

My plan was to split it up into cases for the different type of intervals and come up with formulas for $|(IA_{n})|$, but I'm finding that very tricky.

Thanks

Hi Fermat,

The stated problem is a bit unclear; you used the absolute value bars to indicate both cardinality and Lebesgue measure? I'm assuming that $|I|$ is the Lebesgue measure of $I$ and $|IA_n|$ is the cardinality of $I\cap A_n$ (with the exception that |IA_n| is $\infty$ if $I\cap A_n$ is infinte). If $I$ is an unbounded interval, $|I| = \infty$ and $|I\cap A_n| = \infty$. So the equation will hold. So suppose $I$ is a bounded interval. There are four possible forms for $I$: $(a,b)$, $[a,b)$, $(a,b]$, and $[a,b]$. I'll do the case when $I = [a,b]$, as the others have similar arguments. When $I = [a,b]$, $|I \cap A_n|$ is no more than $nb - na + 1$ and no less than $nb - na - 3$. Hence, $||I \cap A_n| - (nb - na)| < C$ for some constant $C$ independent of $n$. Thus,

\(\displaystyle \left|\frac{|IA_n|}{n} - |I|\right| < \frac{C}{n}.\)

Since $\frac{C}{n} \to 0$ as $n \to \infty$, we deduce that

\(\displaystyle \lim_{n\to \infty} \frac{|IA_n|}{n} = |I|\).
 
Last edited:
  • #3
Euge said:
Hi Fermat,

The stated problem is a bit unclear; you used the absolute value bars to indicate both cardinality and Lebesgue measure? I'm assuming that $|I|$ is the Lebesgue measure of $I$ and $|IA_n|$ is the cardinality of $I\cap A_n$. If $I$ is an unbounded interval, $|I| = \infty$ and $|I\cap A_n| = \infty$. So the equation will hold. So suppose $I$ is a bounded interval. There are four possible forms for $I$: $(a,b)$, $[a,b)$, $(a,b]$, and $[a,b]$. I'll do the case when $I = [a,b]$, as the others have similar arguments. When $I = [a,b]$, $I \cap A_n$ has no more than $nb - na + 1$ elements and no less than $nb - na - 3$ elements. Hence, $||I \cap A_n| - (nb - na)| < C$ for some constant $C$ independent of $n$. Thus,

\(\displaystyle \left|\frac{|IA_n|}{n} - |I|\right| < \frac{C}{n}.\)

Since $\frac{C}{n} \to 0$ as $n \to \infty$, we deduce that

\(\displaystyle \lim_{n\to \infty} \frac{|IA_n|}{n} = |I|\).

Thanks very much. Can I ask how you got the bounds $nb - na + 1$ and $nb - na - 3$, and how I obtain bounds for the other sorts of intervals. Thanks
 
  • #4
Fermat said:
Thanks very much. Can I ask how you got the bounds $nb - na + 1$ and $nb - na - 3$, and how I obtain bounds for the other sorts of intervals. Thanks

When $I = [a, b]$, $I\cap A_n$ is the set of rationals of the form $\frac{k}{n}$ which satisfy $a \le \frac{k}{n} \le b$, i.e., $na \le k \le nb$. So $|I \cap A_n|$ is the number of integers $k$ in the interval $[na,nb]$. The least integer in $[na,nb]$ is either $na$ or $\lfloor na \rfloor + 1$ (depending on whether or not $na$ is an integer) and similarly the greatest integer in $[na,nb]$ is either $nb$ or $\lfloor nb \rfloor - 1$. Hence $|I \cap A_n|$ is either $nb - na + 1$, $\lfloor nb \rfloor - na$, $nb - \lfloor na \rfloor$, or $\lfloor nb \rfloor - \lfloor na \rfloor - 2$. Each of these numbers are between $nb - na - 3$ and $nb - na + 1$ since $\lfloor nb \rfloor \in (nb - 1, nb]$ and $\lfloor na \rfloor \in (na - 1, na]$. Therefore $|I \cap A_n|$ is no more than $nb - na + 1$ than $nb - na + 3$.

The other cases are dealt with in nearly the same way.
 
  • #5
for your question! I understand your approach of breaking down the problem into different cases for different types of intervals. However, there is a simpler and more general approach to proving this statement.

First, let's define the cardinality of a set $S$ as the number of elements in that set, denoted by $|S|$. In this case, we are interested in the cardinality of an interval $I$, which is the number of real numbers contained within that interval.

Next, let's define the set $IA_{n}$ as the intersection of the interval $I$ and the set $A_{n}$. In other words, $IA_{n}$ is the set of all real numbers that are both in the interval $I$ and in the set $A_{n}$. We can write this as $IA_{n} = \{x \in I | x = \frac{k}{n}, k \in \mathbb{Z}\}$.

Now, as $n$ tends to infinity, the set $A_{n}$ becomes denser and denser, meaning that there are more and more real numbers in $A_{n}$ that can be expressed as $\frac{k}{n}$ for some integer $k$. As a result, the intersection $IA_{n}$ will also become denser and closer to the interval $I$ as $n$ tends to infinity.

This leads us to the key idea that the limit of $\frac{1}{n}|(IA_{n})|$ as $n$ tends to infinity is equal to the cardinality of the interval $I$, or $|I|$. This is because as $n$ tends to infinity, the ratio of $|(IA_{n})|$ to $n$ will approach the cardinality of $I$, which is the number of real numbers in $I$.

To prove this statement, we can use the concept of limits. By definition, the limit of a sequence $a_{n}$ is a number $L$ such that for any small positive number $\epsilon$, there exists a positive integer $N$ such that for all $n > N$, the value of $a_{n}$ will be within $\epsilon$ distance from $L$. In other words, as $n$ gets larger and larger, the value of $a_{n}$ gets closer and closer to $L$.

In our case, as $
 

Related to Cardinality of an interval as a limit

1. What is the definition of cardinality of an interval as a limit?

The cardinality of an interval as a limit is the number of elements that the interval approaches as it gets closer and closer to its limit. It is a measure of the size or magnitude of the interval.

2. How is the cardinality of an interval as a limit calculated?

The cardinality of an interval as a limit is calculated by taking the limit of the ratio of the length of the interval to the distance between the two endpoints of the interval. This is also known as the density of the interval.

3. What is the significance of the cardinality of an interval as a limit in mathematics?

The cardinality of an interval as a limit is a fundamental concept in calculus and real analysis. It helps to determine the behavior of functions and is used to define concepts such as continuity and differentiability.

4. How does the cardinality of an interval as a limit relate to other concepts like infinity and infinitesimal?

The cardinality of an interval as a limit is closely related to the concepts of infinity and infinitesimal. In the context of limits, it represents the finite or infinite size of the interval as it approaches a certain point. Infinity and infinitesimal are used to describe the behavior of the interval as it approaches its limit.

5. Can the cardinality of an interval as a limit ever be undefined?

Yes, the cardinality of an interval as a limit can be undefined in certain cases. For example, if the interval has a limit of infinity or infinitesimal, the cardinality cannot be determined. Additionally, if the limit of the ratio of the length of the interval to the distance between the endpoints does not exist, the cardinality will also be undefined.

Similar threads

  • Calculus and Beyond Homework Help
Replies
2
Views
210
  • Topology and Analysis
Replies
2
Views
2K
Replies
4
Views
381
Replies
11
Views
1K
Replies
1
Views
962
  • Math POTW for University Students
Replies
3
Views
606
  • Calculus and Beyond Homework Help
Replies
5
Views
903
  • Topology and Analysis
Replies
4
Views
2K
Replies
16
Views
2K
  • Topology and Analysis
Replies
4
Views
1K
Back
Top